¿Cómo es el magnetismo un resultado de la relatividad especial?

Escucho esto mucho y obviamente sé sobre el video de Veritasium. Cuando la corriente no fluye a través del alambre, sabemos que una carga positiva no sería repelida. Cuando la corriente fluye a través del cable, la carga positiva estacionaria no experimenta repulsión como lo indica la fuerza de Lorentz; pero seguramente, los electrones se contraerían con respecto a los iones positivos en reposo en el marco de referencia de la carga positiva estacionaria a medida que se mueven en relación con él (conocemos el caso predeterminado en el que no fluía corriente, las cargas tenían una distribución igual a causa 0 repulsión, por lo que ahora no pueden ser); Entonces, de acuerdo con la relatividad especial, ¿atraería? Aquí es donde radica la dicotomía para mí.

Además, realmente no entiendo a la gente que dice que el magnetismo es completamente un producto de la relatividad especial, porque ¿qué pasa con el magnetismo intrínseco (es decir, el giro y el momento angular orbital) en los imanes de barra donde todavía se aplica la fuerza de Lorentz?

También he visto el canal, el tipo trató de convencerte aplicando una contracción de longitud en el marco de referencia de electrones y luego de protones, habrá una fuerza magnética en el trabajo.

Respuestas (3)

El magnetismo no es únicamente un efecto relativista.

El punto clave que la gente está tratando de hacer cuando habla de esto es que el campo eléctrico mi y campo magnético B no son cantidades relativistamente invariantes: diferentes observadores medirán diferentes valores de ellas. El ejemplo más simple es una sola carga eléctrica; en su marco de reposo no hay campo magnético, pero genera un campo magnético en un marco donde se mueve.

En el caso de una carga, hay un marco donde no hay campo magnético. Sin embargo, en un sistema más complicado, como dos cargas que se mueven una hacia la otra, o un imán permanente, no existe una transformación de marco que pueda eliminar el campo magnético.

Responderé a su pregunta brindándole el cálculo concreto del campo magnético visto por un observador cuando una carga puntual se mueve con una velocidad v , y para simplificar lo haré un camino en línea recta.

En primer lugar, el campo electromagnético viene dado por un rango 2 tensor, el tensor EM F m v . Al ser un tensor, bajo una transformación de Lorentz, se transforma como

F m v = X m X α X v X β F α β

Tomando la transformación de Lorentz más simple, un impulso a lo largo del X -eje, puede ver fácilmente que los componentes del campo eléctrico y magnético, en el marco k mezclar de esta manera

mi X = mi X B X = B X mi y = γ ( mi y β B z ) B y = γ ( B y + β mi z ) mi z = γ ( mi z + β B y ) B z = γ ( B z β mi y )

De esto encontramos el hecho esclarecedor de que en un marco donde la carga se mueve, medirá un campo magnético así como un campo eléctrico. Supongamos, de hecho, que la carga q , centrado en el marco k , se mueve a lo largo de la X -eje con una velocidad v , y la distancia de máxima aproximación al observador es b . Además, suponga que los dos marcos coinciden en el tiempo t = t = 0 . Nombre el punto en el que el observado mide los campos en su marco PAG que tiene coordenadas en el marco móvil

PAG = ( v t , b , 0 )
y es a distancia
r = b 2 + ( v t ) 2
El campo eléctrico en el marco de la carga es claramente

mi X = q v t r 3 mi y = q b r 3 mi z = 0

el campo magnético es cero en todas partes. Usando la transformación de Lorentz podemos escribir estos campos en la coordenada del marco k

mi X = q γ v t ( b 2 + γ 2 v 2 t 2 ) 3 / 2 mi y = q b ( b 2 + γ 2 v 2 t 2 ) 3 / 2

Luego, al usar la transformación de campo encontrada antes de obtener el resultado de la búsqueda

mi X = mi X = q γ v t ( b 2 + γ 2 v 2 t 2 ) 3 / 2 mi y = γ mi y = γ q b ( b 2 + γ 2 v 2 t 2 ) 3 / 2 B z = γ β mi y = β mi y = β γ q b ( b 2 + γ 2 v 2 t 2 ) 3 / 2

Y aquí tienes

Una persona en un marco k en reposo con respecto a una carga en movimiento uniforme, cuyo marco es k , medirá un campo magnético así como un campo eléctrico.

Se puede encontrar un tratamiento más profundo del tema en el libro de Jackson, ¡obviamente!

Si observa la relación de fuerza de Lorentz clásica, notará que la fuerza de Lorentz depende de la velocidad. Un ejemplo simple es un haz de protones o α haz de partículas. En un marco estacionario (o marco de laboratorio), el haz se mueve muy rápido, por lo tanto, hay una gran fuerza magnética que mantiene el haz unido. Pero en el marco de la α partículas, las fuerzas eléctricas deberían ser dominantes y deberían separarse. Esta fue la paradoja que llevó a Einstein a la relatividad.

Que haya una transformación de lorentz. X X' . Dónde X son las coordenadas del espacio-tiempo del marco de laboratorio y X' son las coordenadas espacio-temporales de un marco que se mueve con el haz de partículas. La transformación se puede expresar como

X m = Λ v m X v
El cuatro potencial electromagnético también se transformará de acuerdo con la transformación de Lorentz.
A m = Λ v m A v
A partir de aquí se pueden determinar los campos eléctrico y magnético. Cuando se considera esta transformación, las fuerzas netas serán las mismas porque los campos ahora son invariantes de Lorentz.

Espero que esto ayude.